Primes $p$ of the form $p=frac7^q+17^q-n^2+1$, I ask some questionsPrimes of the form $frac7^q+17^m+1$, where q is a positive integer and $m=q-n^2$Is it possible for a number in form $1987^k-1$ to end with 1987 zeros? Also few questions about number theory in general.Some questions about Goldbach's conjectureWhat numbers can be expressed with the following expression?Generating pairs of primes from the 2 previous primes.Find all primes of the form $3^n -1$Proving there are infinite primes of some form $a+nd$Finding primes by summing the first n consecutive primesPrimes in intervals of special form and search for the minimal $k$ (if it exists)Can you propose a conjectural $textUpper bound(x)$ for the counting function of a sequence of primes arising from the Eratosthenes sieve?The hunting of “missing primes”

What reasons are there for a Capitalist to oppose a 100% inheritance tax?

How much of data wrangling is a data scientist's job?

If human space travel is limited by the G force vulnerability, is there a way to counter G forces?

What is the most common color to indicate the input-field is disabled?

What exploit Are these user agents trying to use?

Why is it a bad idea to hire a hitman to eliminate most corrupt politicians?

What killed these X2 caps?

How can saying a song's name be a copyright violation?

Ambiguity in the definition of entropy

Am I breaking OOP practice with this architecture?

How to tell a function to use the default argument values?

Why is this clock signal connected to a capacitor to gnd?

Why can't we play rap on piano?

Intersection Puzzle

What's the in-universe reasoning behind sorcerers needing material components?

What method can I use to design a dungeon difficult enough that the PCs can't make it through without killing them?

Plagiarism or not?

Assassin's bullet with mercury

Is it acceptable for a professor to tell male students to not think that they are smarter than female students?

What mechanic is there to disable a threat instead of killing it?

How can I determine if the org that I'm currently connected to is a scratch org?

Venezuelan girlfriend wants to travel the USA to be with me. What is the process?

Forgetting the musical notes while performing in concert

How dangerous is XSS?



Primes $p$ of the form $p=frac7^q+17^q-n^2+1$, I ask some questions


Primes of the form $frac7^q+17^m+1$, where q is a positive integer and $m=q-n^2$Is it possible for a number in form $1987^k-1$ to end with 1987 zeros? Also few questions about number theory in general.Some questions about Goldbach's conjectureWhat numbers can be expressed with the following expression?Generating pairs of primes from the 2 previous primes.Find all primes of the form $3^n -1$Proving there are infinite primes of some form $a+nd$Finding primes by summing the first n consecutive primesPrimes in intervals of special form and search for the minimal $k$ (if it exists)Can you propose a conjectural $textUpper bound(x)$ for the counting function of a sequence of primes arising from the Eratosthenes sieve?The hunting of “missing primes”













0












$begingroup$


In a previous post I asked for primes $p$ of the form: $$p=frac7^q+17^q-n^2+1,$$ where $q$ and $n$ are positive integers.



The solutions found up to $q=5000$ are $[1,17,4]$, $[8,24,4]$, $[2,38,6]$ and $[4,148,12]$. In brackets the second number is $q$, the first number is $q-n^2$ and the third number is $n$.



Do you believe that the next solution is extremely huge or even far beyond the programs and calculators capacities?

Do you believe that the number of these primes is not infinite?

Do you believe that a prime of this type exists such that $q-n^2neq 2^k$ where $k$ is an integer $geqslant 0$?



Here is the link to the previous question










share|cite|improve this question











$endgroup$











  • $begingroup$
    Why are there 3 numbers in the square brackets?
    $endgroup$
    – Pink Panther
    Mar 21 at 8:05










  • $begingroup$
    @Pink Panter the second number is q, the first number is $q-n^2$ and the third number is $n$.
    $endgroup$
    – homunculus
    Mar 21 at 8:07















0












$begingroup$


In a previous post I asked for primes $p$ of the form: $$p=frac7^q+17^q-n^2+1,$$ where $q$ and $n$ are positive integers.



The solutions found up to $q=5000$ are $[1,17,4]$, $[8,24,4]$, $[2,38,6]$ and $[4,148,12]$. In brackets the second number is $q$, the first number is $q-n^2$ and the third number is $n$.



Do you believe that the next solution is extremely huge or even far beyond the programs and calculators capacities?

Do you believe that the number of these primes is not infinite?

Do you believe that a prime of this type exists such that $q-n^2neq 2^k$ where $k$ is an integer $geqslant 0$?



Here is the link to the previous question










share|cite|improve this question











$endgroup$











  • $begingroup$
    Why are there 3 numbers in the square brackets?
    $endgroup$
    – Pink Panther
    Mar 21 at 8:05










  • $begingroup$
    @Pink Panter the second number is q, the first number is $q-n^2$ and the third number is $n$.
    $endgroup$
    – homunculus
    Mar 21 at 8:07













0












0








0


1



$begingroup$


In a previous post I asked for primes $p$ of the form: $$p=frac7^q+17^q-n^2+1,$$ where $q$ and $n$ are positive integers.



The solutions found up to $q=5000$ are $[1,17,4]$, $[8,24,4]$, $[2,38,6]$ and $[4,148,12]$. In brackets the second number is $q$, the first number is $q-n^2$ and the third number is $n$.



Do you believe that the next solution is extremely huge or even far beyond the programs and calculators capacities?

Do you believe that the number of these primes is not infinite?

Do you believe that a prime of this type exists such that $q-n^2neq 2^k$ where $k$ is an integer $geqslant 0$?



Here is the link to the previous question










share|cite|improve this question











$endgroup$




In a previous post I asked for primes $p$ of the form: $$p=frac7^q+17^q-n^2+1,$$ where $q$ and $n$ are positive integers.



The solutions found up to $q=5000$ are $[1,17,4]$, $[8,24,4]$, $[2,38,6]$ and $[4,148,12]$. In brackets the second number is $q$, the first number is $q-n^2$ and the third number is $n$.



Do you believe that the next solution is extremely huge or even far beyond the programs and calculators capacities?

Do you believe that the number of these primes is not infinite?

Do you believe that a prime of this type exists such that $q-n^2neq 2^k$ where $k$ is an integer $geqslant 0$?



Here is the link to the previous question







number-theory prime-numbers






share|cite|improve this question















share|cite|improve this question













share|cite|improve this question




share|cite|improve this question








edited Mar 21 at 8:50









Andrews

1,2812422




1,2812422










asked Mar 21 at 8:00









homunculushomunculus

1869




1869











  • $begingroup$
    Why are there 3 numbers in the square brackets?
    $endgroup$
    – Pink Panther
    Mar 21 at 8:05










  • $begingroup$
    @Pink Panter the second number is q, the first number is $q-n^2$ and the third number is $n$.
    $endgroup$
    – homunculus
    Mar 21 at 8:07
















  • $begingroup$
    Why are there 3 numbers in the square brackets?
    $endgroup$
    – Pink Panther
    Mar 21 at 8:05










  • $begingroup$
    @Pink Panter the second number is q, the first number is $q-n^2$ and the third number is $n$.
    $endgroup$
    – homunculus
    Mar 21 at 8:07















$begingroup$
Why are there 3 numbers in the square brackets?
$endgroup$
– Pink Panther
Mar 21 at 8:05




$begingroup$
Why are there 3 numbers in the square brackets?
$endgroup$
– Pink Panther
Mar 21 at 8:05












$begingroup$
@Pink Panter the second number is q, the first number is $q-n^2$ and the third number is $n$.
$endgroup$
– homunculus
Mar 21 at 8:07




$begingroup$
@Pink Panter the second number is q, the first number is $q-n^2$ and the third number is $n$.
$endgroup$
– homunculus
Mar 21 at 8:07










0






active

oldest

votes












Your Answer





StackExchange.ifUsing("editor", function ()
return StackExchange.using("mathjaxEditing", function ()
StackExchange.MarkdownEditor.creationCallbacks.add(function (editor, postfix)
StackExchange.mathjaxEditing.prepareWmdForMathJax(editor, postfix, [["$", "$"], ["\\(","\\)"]]);
);
);
, "mathjax-editing");

StackExchange.ready(function()
var channelOptions =
tags: "".split(" "),
id: "69"
;
initTagRenderer("".split(" "), "".split(" "), channelOptions);

StackExchange.using("externalEditor", function()
// Have to fire editor after snippets, if snippets enabled
if (StackExchange.settings.snippets.snippetsEnabled)
StackExchange.using("snippets", function()
createEditor();
);

else
createEditor();

);

function createEditor()
StackExchange.prepareEditor(
heartbeatType: 'answer',
autoActivateHeartbeat: false,
convertImagesToLinks: true,
noModals: true,
showLowRepImageUploadWarning: true,
reputationToPostImages: 10,
bindNavPrevention: true,
postfix: "",
imageUploader:
brandingHtml: "Powered by u003ca class="icon-imgur-white" href="https://imgur.com/"u003eu003c/au003e",
contentPolicyHtml: "User contributions licensed under u003ca href="https://creativecommons.org/licenses/by-sa/3.0/"u003ecc by-sa 3.0 with attribution requiredu003c/au003e u003ca href="https://stackoverflow.com/legal/content-policy"u003e(content policy)u003c/au003e",
allowUrls: true
,
noCode: true, onDemand: true,
discardSelector: ".discard-answer"
,immediatelyShowMarkdownHelp:true
);



);













draft saved

draft discarded


















StackExchange.ready(
function ()
StackExchange.openid.initPostLogin('.new-post-login', 'https%3a%2f%2fmath.stackexchange.com%2fquestions%2f3156500%2fprimes-p-of-the-form-p-frac7q17q-n21-i-ask-some-questions%23new-answer', 'question_page');

);

Post as a guest















Required, but never shown

























0






active

oldest

votes








0






active

oldest

votes









active

oldest

votes






active

oldest

votes















draft saved

draft discarded
















































Thanks for contributing an answer to Mathematics Stack Exchange!


  • Please be sure to answer the question. Provide details and share your research!

But avoid


  • Asking for help, clarification, or responding to other answers.

  • Making statements based on opinion; back them up with references or personal experience.

Use MathJax to format equations. MathJax reference.


To learn more, see our tips on writing great answers.




draft saved


draft discarded














StackExchange.ready(
function ()
StackExchange.openid.initPostLogin('.new-post-login', 'https%3a%2f%2fmath.stackexchange.com%2fquestions%2f3156500%2fprimes-p-of-the-form-p-frac7q17q-n21-i-ask-some-questions%23new-answer', 'question_page');

);

Post as a guest















Required, but never shown





















































Required, but never shown














Required, but never shown












Required, but never shown







Required, but never shown

































Required, but never shown














Required, but never shown












Required, but never shown







Required, but never shown







Popular posts from this blog

Lowndes Grove History Architecture References Navigation menu32°48′6″N 79°57′58″W / 32.80167°N 79.96611°W / 32.80167; -79.9661132°48′6″N 79°57′58″W / 32.80167°N 79.96611°W / 32.80167; -79.9661178002500"National Register Information System"Historic houses of South Carolina"Lowndes Grove""+32° 48' 6.00", −79° 57' 58.00""Lowndes Grove, Charleston County (260 St. Margaret St., Charleston)""Lowndes Grove"The Charleston ExpositionIt Happened in South Carolina"Lowndes Grove (House), Saint Margaret Street & Sixth Avenue, Charleston, Charleston County, SC(Photographs)"Plantations of the Carolina Low Countrye

random experiment with two different functions on unit interval Announcing the arrival of Valued Associate #679: Cesar Manara Planned maintenance scheduled April 23, 2019 at 00:00UTC (8:00pm US/Eastern)Random variable and probability space notionsRandom Walk with EdgesFinding functions where the increase over a random interval is Poisson distributedNumber of days until dayCan an observed event in fact be of zero probability?Unit random processmodels of coins and uniform distributionHow to get the number of successes given $n$ trials , probability $P$ and a random variable $X$Absorbing Markov chain in a computer. Is “almost every” turned into always convergence in computer executions?Stopped random walk is not uniformly integrable

How should I support this large drywall patch? Planned maintenance scheduled April 23, 2019 at 00:00UTC (8:00pm US/Eastern) Announcing the arrival of Valued Associate #679: Cesar Manara Unicorn Meta Zoo #1: Why another podcast?How do I cover large gaps in drywall?How do I keep drywall around a patch from crumbling?Can I glue a second layer of drywall?How to patch long strip on drywall?Large drywall patch: how to avoid bulging seams?Drywall Mesh Patch vs. Bulge? To remove or not to remove?How to fix this drywall job?Prep drywall before backsplashWhat's the best way to fix this horrible drywall patch job?Drywall patching using 3M Patch Plus Primer